Physio Finals (24-25) Flashcards

1
Q
  1. The Gibbs-Donnan effect is the increase in osmotically active particles in the cell due to an impermeant intracellular molecule which is:
    A. Potassium
    B. Proteins
    C. Water
    D. Chloride
A

B. Proteins

How well did you know this?
1
Not at all
2
3
4
5
Perfectly
2
Q
  1. In skeletal muscle, this stimulates the RYR (ryanodine) receptors to release calcium from the terminal cisternae of the sarcoplasmic reticulum
    A. Conformational change of DHPR receptors
    B. Increased activity of SERCA
    C. Release of acetylcholine from presynaptic terminal
    D. Entry of ECF calcium
A

Conformational change of DHPR receptors

How well did you know this?
1
Not at all
2
3
4
5
Perfectly
3
Q
  1. The sinoatrial node is the pacemaker of the heart for this reason
    A. It has the fastest spontaneous depolarization
    B. It is the largest cell with spontaneous depolarization
    C. It is located in the right atrium
    D. It connects to the AV node
A

It has the fastest spontaneous depolarization

How well did you know this?
1
Not at all
2
3
4
5
Perfectly
4
Q
  1. Smooth muscles particularly the single unit type are able to contract as a unit due to the rapid transmission of action potentials between cells through
    A. Gap junctions
    B. Zona occludens
    C. Terminal varicosities
    D. Tight junctions
A

Gap junctions

How well did you know this?
1
Not at all
2
3
4
5
Perfectly
5
Q
  1. The energy for one of the following transport systems comes from the direct utilization or hydrolysis of ATP
    A. Passive transport
    B. Primary active transport
    C. Secondary active transport
    D. Facilitated diffusion
A

Primary active transport

How well did you know this?
1
Not at all
2
3
4
5
Perfectly
6
Q
  1. Cardiac muscle cells are able to modulate the force of contraction through this mechanism
    A. Recruitment of more motor units
    B. Tetanic contractions
    C. Increase in ATP binding to myosin head
    D. Varying amount of extracellular Ca2+ that enters
A

Varying amount of extracellular Ca2+ that enters

How well did you know this?
1
Not at all
2
3
4
5
Perfectly
7
Q
  1. A change in the ECF value of one of the following ions will affect membrane voltage
    A. Potassium
    B. Sodium
    C. Calcium
    D. Chloride
A

Sodium

How well did you know this?
1
Not at all
2
3
4
5
Perfectly
8
Q
  1. The activity of the following will result to the relaxation of cardiac muscles EXCEPT:
    A. Sarcolemmal Ca2+ pump
    B. SERCA
    C. Voltage-gated L type channel
    D. 3Na+1Ca2+ antiporter
A

Voltage-gated L type channel

How well did you know this?
1
Not at all
2
3
4
5
Perfectly
9
Q
  1. Which of the following binds to nuclear receptors to affect gene expression?
    A. Acetylcholine
    B. Epinephrine
    C. Atrial natriuretic peptide
    D. Steroid hormones
A

Steroid hormones

How well did you know this?
1
Not at all
2
3
4
5
Perfectly
10
Q
  1. Which of these sensory afferent fibers of the muscle spindles is sensitive to both amount and rate of stretch and is therefore sensitive to transient stretches such as when a doctor elicits the knee jerk reflex by tapping the patellar tendon?
    A. Group II
    B. Group Ia
    C. Group III
    D. Group Ib
A

Group Ia

How well did you know this?
1
Not at all
2
3
4
5
Perfectly
11
Q
  1. Which of the following causes the movement of water between ICF and ECF?
    A. Osmotic pressure difference
    B. Membrane permeability to water
    C. Amounts of water ingested
    D. Number of aquaporins present
A

Osmotic pressure difference

How well did you know this?
1
Not at all
2
3
4
5
Perfectly
12
Q
  1. A decrease in one of the following will most affect plasma oncotic pressure
    A. Membrane voltage
    B. ATP
    C. Proteins
    D. Sodium
A

Proteins

How well did you know this?
1
Not at all
2
3
4
5
Perfectly
13
Q
  1. In smooth muscles, calcium can be released from inositol triphosphate (InsP3)-gated Ca2+ channel in response to
    A. Hormones binding to receptors
    B. Action potentials
    C. ATP binding to myosin heads
    D. ECF Ca2+ entering the cell
A

Hormones binding to receptors

How well did you know this?
1
Not at all
2
3
4
5
Perfectly
14
Q
  1. The most important factor in the onset of skeletal muscle fatigue is
    A. CNS conditioning
    B. Depletion of energy stores
    C. Accumulation of metabolic by-products
    D. Increase in pH in the environment
A

Accumulation of metabolic by-products

How well did you know this?
1
Not at all
2
3
4
5
Perfectly
15
Q
  1. In skeletal muscles, one of the following causes myosin heads to dissociate from the actin binding sites
    A. Anticipation of next powerstroke
    B. Decrease in the amount of ATP
    C. Binding of new ATP molecule to myosin head
    D. Increase in myoplasmic calcium
A

Binding of new ATP molecule to myosin head

How well did you know this?
1
Not at all
2
3
4
5
Perfectly
16
Q
  1. Which of the following pairs is correct regarding the neurotransmitters released by sympathetic and parasympathetic fibers?
    A. Preganglionic neurons of both sympathetic and parasympathetic are adrenergic
    B. Majority of postganglionic sympathetic fibers are cholinergic
    C. Postganglionic parasympathetic fibers are cholinergic
    D. Postganglionic sympathetic fibers to the piloerector muscles are adrenergic
A

Postganglionic parasympathetic fibers are cholinergic

How well did you know this?
1
Not at all
2
3
4
5
Perfectly
17
Q
  1. Which of the following organelles is correctly matched with its function?
    A. Mitochondria – site of oxidative phosphorylation
    B. Smooth endoplasmic reticulum – protein synthesis
    C. Rough endoplasmic reticulum – contains proteases, lipases, and nucleases
    D. Golgi apparatus – detoxification and lipid synthesis
A

Mitochondria – site of oxidative phosphorylation

How well did you know this?
1
Not at all
2
3
4
5
Perfectly
18
Q
  1. This second messenger activates ligand-activated Ca2+ channels to release Ca2+ from the endoplasmic reticulum in smooth muscles
    A. Leukotrienes
    B. Inositol triphosphate
    C. cAMP
    D. Nitric oxide
A

Inositol triphosphate

How well did you know this?
1
Not at all
2
3
4
5
Perfectly
19
Q
  1. One of the following causes the relaxation of skeletal muscles
    A. Decrease in amounts of available ATP
    B. Hydrolysis of ATP to ADP + Pi
    C. Resequestration of Ca2+ into the SR
    D. Powerstroke of actin and myosin filaments
A

Resequestration of Ca2+ into the SR

How well did you know this?
1
Not at all
2
3
4
5
Perfectly
20
Q
  1. Stretch of cardiac muscles can also increase the force of contraction because
    A. More actin and myosin interactions due to decreased space between them
    B. Increased sensitivity to Ca2+
    C. Both
    D. Neither
A

More actin and myosin interactions due to decreased space between them

How well did you know this?
1
Not at all
2
3
4
5
Perfectly
21
Q
  1. Which of the following is an adrenergic/sympathetic response?
    A. Pupillary constriction
    B. Increase intestinal secretion
    C. Decreased heart rate
    D. Bronchial muscle relaxation
A

Bronchial muscle relaxation

How well did you know this?
1
Not at all
2
3
4
5
Perfectly
22
Q
  1. What is the role of Ca2+ in smooth muscle contraction?
    A. Binds to Troponin C to expose actin binding sites
    B. Binds to calmodulin to activate myosin light chain kinase
    C. Cause conformational change in tropomyosin
    D. Cause slow cycling of actin and myosin
A

Binds to calmodulin to activate myosin light chain kinase

How well did you know this?
1
Not at all
2
3
4
5
Perfectly
23
Q
  1. Protein kinase C is activated in the presence of which of the following second messenger?
    A. Inositol triphosphate
    B. cAMP
    C. Tyrosine kinase
    D. DAG
A

DAG

How well did you know this?
1
Not at all
2
3
4
5
Perfectly
24
Q
  1. Water moves out from the ICF to the ECF with the addition of
    A. Hypotonic NaCl solution to the ECF
    B. Isotonic NaCl solution to the ECF
    C. Hypertonic NaCl solution to the ECF
    D. Isotonic solution to the ICF
A

Hypertonic NaCl solution to the ECF

How well did you know this?
1
Not at all
2
3
4
5
Perfectly
25
Q
  1. The ability of a cell to respond to a signalling molecule with specificity and high affinity depends on the presence of one of the following
    A. Hormones
    B. Second messengers
    C. Receptors
    D. Neurotransmitters
A

Receptors

How well did you know this?
1
Not at all
2
3
4
5
Perfectly
26
Q
  1. Which of the following secretes antibodies that are reactive agents against foreign antigens?
    A. B lymphocytes
    B. T helper lymphocytes
    C. T cytotoxic lymphocytes
    D. T suppressor lymphocytes
A

B lymphocytes

How well did you know this?
1
Not at all
2
3
4
5
Perfectly
27
Q
  1. Selective phagocytosis to prevent ingestion of normal cells of the body by neutrophils and macrophages takes place by
    A. Tissues and cells having smooth surfaces
    B. Absence of protective coat on cell structures
    C. Antibodies targeting normal tissues
    D. Non selective opsonisation by C3
A

Tissues and cells having smooth surfaces

How well did you know this?
1
Not at all
2
3
4
5
Perfectly
28
Q
  1. Agglutination: negative Anti-A, positive Anti-B
    A. Type O
    B. Type A
    C. Type B
    D. Type AB
A

Type A

How well did you know this?
1
Not at all
2
3
4
5
Perfectly
29
Q
  1. These are greatly enhanced forms of B lymphocytes, once exposed to specific antigen cause a much rapid and potent antibody response
    A. Plasmablast
    B. Lymphoblast
    C. Memory cells
    D. Antigen presenting cells
A

Memory cells

How well did you know this?
1
Not at all
2
3
4
5
Perfectly
30
Q
  1. Inflamed tissues releases cytokines which causes endothelial cells to secret selectins and ICAM-1 which reacts with neutrophils’ integrin causing adhesion
    A. Diapedesis
    B. Margination
    C. Phagocytosis
    D. Chemotaxis
A

Margination

How well did you know this?
1
Not at all
2
3
4
5
Perfectly
31
Q
  1. Formed during the primary response
    A. IgG
    B. IgM
    C. IgE
    D. IgA
A

IgM

How well did you know this?
1
Not at all
2
3
4
5
Perfectly
32
Q
  1. The most common form of haemoglobin in the adult human being is a combination of
    A. Two alpha chains and two gamma chains
    B. Two alpha chains and two beta chains
    C. Two alpha chains and two delta chains
    D. Two beta chains and two delta chains
A

Two alpha chains and two beta chains

How well did you know this?
1
Not at all
2
3
4
5
Perfectly
33
Q
  1. Which of the following is a direct action of antibodies when they cover the toxic sites of the antigen?
    A. Agglutination
    B. Precipitation
    C. Neutralization
    D. Lysis
A

Neutralization

How well did you know this?
1
Not at all
2
3
4
5
Perfectly
34
Q
  1. After age 20, red blood cells are produced in the following
    A. Iliac
    B. Spleen
    C. Bone marrow
    D. Lymph nodes
A

Iliac

How well did you know this?
1
Not at all
2
3
4
5
Perfectly
35
Q
  1. Transplant between identical twins are called
    A. Autografts
    B. Isografts
    C. Allografts
    D. Xenografts
A

Isografts

How well did you know this?
1
Not at all
2
3
4
5
Perfectly
36
Q
  1. Direct attack cells that binds to invasive organisms which secretes perforins
    A. T helper cells
    B. T cytotoxic cells
    C. T suppressor cells
    D. B cells
A

T cytotoxic cells

How well did you know this?
1
Not at all
2
3
4
5
Perfectly
37
Q
  1. Which of the following have the greatest diversity?
    A. B lymphocytes
    B. T helper lymphocytes
    C. T cytotoxic lymphocytes
    D. T suppressor lymphocytes
A

T helper lymphocytes

How well did you know this?
1
Not at all
2
3
4
5
Perfectly
38
Q
  1. This portion of the antibodies attaches to a particular type of antigen
    A. Variable portion
    B. Constant portion
    C. Light chain
    D. Heavy chain
A

Variable portion

How well did you know this?
1
Not at all
2
3
4
5
Perfectly
39
Q
  1. Which type of WBC has the longest life span?
    A. Neutrophils
    B. Lymphocytes
    C. Monocytes
    D. Basophils
A

Lymphocytes

How well did you know this?
1
Not at all
2
3
4
5
Perfectly
40
Q
  1. Most numerous T cells and major regulator of the immune system forming lymphokines
    A. T helper cells
    B. T cytotoxic cells
    C. T suppressor cells
    D. B cells
A

T helper cells

How well did you know this?
1
Not at all
2
3
4
5
Perfectly
41
Q
  1. Of the total WBCs in the blood, which type has the highest percentage?
    A. Neutrophils
    B. Lymphocytes
    C. Monocytes
    D. Basophils
A

Neutrophils

How well did you know this?
1
Not at all
2
3
4
5
Perfectly
42
Q
  1. Which of the following are examples of acquired immunity?
    A. Immunization
    B. Non-IgE allergy
    C. Asthma
    D. Parasitism
A

Immunization

How well did you know this?
1
Not at all
2
3
4
5
Perfectly
43
Q
  1. Which of the following is/are responsible for humoral immunity?
    A. T cytotoxic lymphocytes
    B. T helper lymphocytes
    C. T suppressor lymphocytes
    D. B lymphocytes
A

B lymphocytes

How well did you know this?
1
Not at all
2
3
4
5
Perfectly
44
Q
  1. Which of the following best describes the human body’s innate immunity?
    A. Develop after the body is first attacked by bacteria, viruses or toxins
    B. Often weeks or months are required for the immunity to develop
    C. Presence in the blood of certain substances and cells that attaches to foreign organisms and destroy them
    D. Powerful and specific response to an individual invading agent
A

Presence in the blood of certain substances and cells that attaches to foreign organisms and destroy them

How well did you know this?
1
Not at all
2
3
4
5
Perfectly
45
Q
  1. This portion of the antibodies determines the ease by which the antibodies pass through membrane
    A. Variable portion
    B. Constant portion
    C. Light chain
    D. Heavy chain
A

Constant portion

How well did you know this?
1
Not at all
2
3
4
5
Perfectly
46
Q
  1. This portion of the antibodies determines adherence to specific structures within the tissues
    A. Variable portion
    B. Constant portion
    C. Light chain
    D. Heavy chain
A

Constant portion

How well did you know this?
1
Not at all
2
3
4
5
Perfectly
47
Q
  1. Vitamin B12 and folic acid are most essential in which stage of erythropoiesis?
    A. Colony formation
    B. Differentiation
    C. Growth
    D. Maturation
A

Differentiation

How well did you know this?
1
Not at all
2
3
4
5
Perfectly
48
Q
  1. When Iron (Fe2+) combines with a beta globulin in the blood plasma and transported to tissue cells
    A. Ferritin
    B. Transferrin
    C. Apotransferrin
    D. Apoferritin
A

B. Transferrin

How well did you know this?
1
Not at all
2
3
4
5
Perfectly
49
Q
  1. The first cell that can be identified as belonging to the eryhthroblast series
    A. Basophil erythroblast
    B. Polychomatophil erythroblast
    C. Orthochromatic erythroblast
    D. Proerythroblast
A

Proerythroblast

How well did you know this?
1
Not at all
2
3
4
5
Perfectly
50
Q
  1. Preprocessing of T lymphocytes occur in the
    A. Liver
    B. Bone marrow
    C. Thymus
    D. Lymph nodes
A

Thymus

51
Q
  1. Which of the following humoral factors contribute to vasodilatation?
    A. Adenosine
    B. Endothelin
    C. Vasopressin
    D. Angiotensin II
A

Adenosine

52
Q
  1. Which of the following compensatory mechanisms is expected to occur when a person stands up from a supine position?
    A. Increase contractility
    B. Decrease in heart rate
    C. Decrease in cardiac output
    D. Decrease total peripheral resistance
A

Increase contractility

53
Q
  1. Which of the following statements does NOT describe the cardiac function curve?
    A. Cardiac output is dependent on left ventricular end-diastolic pressure
    B. Cardiac output is dependent on central venous pressure
    C. Cardiac output is dependent on right atrial pressure
    D. Cardiac output is dependent on right ventricular end-diastolic pressure
A

Cardiac output is dependent on left ventricular end-diastolic pressure

54
Q
  1. Which of the following receptors would decrease the heart rate when stimulated?
    A. Muscarinic receptors
    B. β2 receptors
    C. β1 receptors
    D. α1 receptors
A

β2 receptors

55
Q
  1. Which of the following intracellular concentrations of ions best correlated with myocardial contractility?
    A. Ca2+
    B. K+
    C. Na+
    D. Mg2+
A

Ca2+

56
Q
  1. Which of the following mechanisms would explain the increase in myocardial contractility when increasing heart rate?
    A. More Ca2+ ions enter the cell during each action potential
    B. Increasing entry of more Ca2+ during the plateau
    C. Inhibiting the Na+-K+ pump
    D. Increasing the storage of Ca2+ by the sarcoplasmic reticulum
A

More Ca2+ ions enter the cell during each action potential

57
Q
  1. Which of the following would demonstrate increase in myocardial contractility by the Frank-Starling relationship?
    A. Decreased cardiac output for a given end-diastolic volume
    B. Increased cardiac output for a given end-diastolic volume
    C. Increased cardiac output for a given end-systolic volume
    D. Decreased cardiac output for a given end-systolic volume
A

Increased cardiac output for a given end-diastolic volume

58
Q
  1. Which of the following conditions will increase output?
    A. Hypertension
    B. Normal inspiration
    C. Severe hypoxemia
    D. Supraventricular tachycardia
A

Normal inspiration

59
Q
  1. Which of the following components is expected to decrease when administering a positive inotropic agent?
    A. Cardiac output
    B. End-systolic volume
    C. Stroke volume
    D. Heart rate
A

End-systolic volume

60
Q
  1. Which of the following parameters is NOT expected during moderate exercise?
    A. Increase in total peripheral resistance
    B. Increase in cardiac output
    C. Increase in heart rate
    D. Increase in pulse pressure
A

Increase in total peripheral resistance

61
Q
  1. Which of the following variables can be used interchangeably to label the x-axis in the cardiac-vascular curve?
    Picture
    A. Pulse pressure
    B. Mean systemic filling pressure
    C. End-systolic volume
    D. End-diastolic volume
A

End-diastolic volume

62
Q
  1. Which of the following statements best describe the compensatory mechanisms during an acute blood loss?
    A. Direction of the secondary changes in pressure is toward the direction of the initiating change
    B. Reduction of vagal tone and enhancement of sympathetic tone
    C. Threshold for stimulation of baroreceptors is at an arterial pressure of 40 mmHg.
    D. NaCl and water excretion is increased
A

Reduction of vagal tone and enhancement of sympathetic tone

63
Q
  1. Which of the following conditions would increase myocardial contractility?
    A. Severe hypoxia
    B. Cholinergic stimulation
    C. Infusion of norepinephrine
    D. inspiration
A

Infusion of norepinephrine

64
Q
  1. Which of the following factors will cause the upward shift as seen in the cardiac function curve?
    Picture
    A. Increased in total peripheral resistance
    B. Increased mean systemic filling pressure
    C. Increased blood volume
    D. Increased contractility
A

Increased contractility

65
Q
  1. Entry of Ca2+ into myocytes is responsible for phase __ of cardiac action potential
    A. 0
    B. 1
    C. 2
    D. 3
A

0

66
Q
  1. Heart rate increases when one of these is present
    A. Faster rate of phase 0
    B. Faster rate of phase 3
    C. Less negative resting membrane potential
    D. Less negative threshold potential
A

Faster rate of phase 0

67
Q
  1. Restoration of ionic concentration during phase 4 is achieved by
    A. Ca2+ influx by 3Na+1Ca2+ antiporter
    B. Ca2+ influx by Ca2+ATPase pump
    C. Na+ efflux by 3Na+Ca2+ antiporter
    D. Na+ efflux by Na+K+ATPase
A

Na+ efflux by Na+K+ATPase

68
Q
  1. Closure of aortic valve is heard best at
    A. 2nd intercostal space left sternal border
    B. 2nd intercostal space right sternal border
    C. 5th intercostal space left midclavicular line
    D. 5th intercostal space right sternal border
A

2nd intercostal space right sternal border

69
Q
  1. Ejection phase of cardiac cycle has one of these characteristics
    A. Left ventricular pressure is low
    B. Left ventricular volume is low
    C. Pulmonic valve is closed
    D. Tricuspid valve is open
A

Left ventricular volume is low

70
Q
  1. The period between closure of semilunar valves and opening of atrioventricular valves is
    A. Isovolumic contraction
    B. Isovolumetric relaxation
    C. Rapid ejection
    D. Rapid filling
A

Isovolumetric relaxation

71
Q
  1. Phase 3 of cardiac action potential is due to
    A. Efflux of K+
    B. Efflux of Na+
    C. Influx of K+
    D. Influx of Na+
A

Efflux of K+

72
Q
  1. Slow response action potential has one of these characteristics
    A. Absent phase 2
    B. Less steep phase 0
    C. More negative phase 4
    D. More distinct phase 3
A

More negative phase 4

73
Q
  1. P wave of the electrocardiogram coincides with ___ phase of cardiac cycle
    A. Atrial systole
    B. Diastasis
    C. Ejection
    D. Filling
A

Atrial systole

74
Q
  1. Stroke volume increases when there is INCREASED:
    A. Afterload
    B. Heart rate
    C. Peripheral vascular resistance
    D. Venous return
A

Heart rate

75
Q
  1. The glomerular filtration barrier is composed of all of the following EXCEPT:
    A. Fenestrated capillary endothelium
    B. Mesangial cells

C. Basement membrane
D. Podocytes

A

Mesangial cells

76
Q
  1. Which of the following will most likely increase GFR? The glomerular filtration rate increases when

A. The afferent arteriolar resistance increases

B. The efferent arteriolar resistance decreases
C. There is an obstruction in urine flow

D. The plasma protein concentration decreases

A

The plasma protein concentration decreases

77
Q
  1. If all other variables are kept constant, what happens to the glomerular filtration rate when arterial blood pressure decreases?

A. Increases
B. Decreases
C. Remains the same

A

Decreases

78
Q
  1. If afferent arteriole radius decreases in response to an increase in blood pressure, then glomerular filtration _____

A. Remains approximately the same
B. Increases
C. Decreases

A

Decreases

79
Q
  1. Which of the following may NOT stimulate the JG cells to increase in renin secretion?
    A. Renal ischemia
    B. Increased amount of Na+ in PCT
    C. Narrowing of afferent arteriole
    D. Decreased amount of Na+ in DCT
A

Increased amount of Na+ in PCT

80
Q
  1. Which of the following nephron segments will the tubular fluid be isotonic to plasma when ADH secretion is high?

A. Ascending limb of the loop of Henle
B. Cortical collecting tubule
C. Medullary collecting tubule
D. Descending limb of the loop of Henle

A

Cortical collecting tubule

81
Q
  1. In the presence of ALDOSTERONE the tubular fluid osmolarity in LETTER E will Picture
    A. Decrease
    B. Be predominantly made up of anions
    C. Remain the same
    D. Increase
A

Decrease

82
Q

A. When tubular fluid volume increases in LETTER A, which of the following will be the correct consequence? Picutre

A. Tubular fluid flow increases

B. Tubular fluid osmolarity in A will increase

C. Tubular fluid oncotic pressure in B will increase

D. Tubular fluid reabsorption in C increases

A

Tubular fluid flow increases

83
Q
  1. What happens if afferent arteriolar resistance increases? GFR will _____ and RBF will
    _____
    A. Increase; increase
    B. Increase; decrease
    C. Decrease; decrease
    D. Decrease; increase
A

Decrease; decrease

84
Q
  1. As tubular flow increases, potassium secretion in the distal tubules and collecting ducts will

A. Prevent Na reabsorption via the Enac channels

B. Prevent the activity of the Na/K ATPase in the basolateral membrane
C. Decrease
D. Increase

A

Increase

85
Q
  1. Decreasing the resistance of the afferent arteriole in the glomerulus of the kidney will decrease which of the following?

A. Glomerular filtration rate
B. Filtration fraction

C. Renal plasma flow

D. Renin release from juxtaglomerular cells

A

Renal plasma flow

86
Q
  1. As blood flows into the afferent arteriole of a newly transplanted kidney, the countercurrent multiplication system will start when:

A. The Na/K ATPase activity in the TAL is activated

B. The ultrafiltrate is delivered into the Bowmans space

C. Tubular fluid flows into the descending thin limb

D. The 1Na1K2Cl symport system in the TAL reabsorbs the solutes

A

The 1Na1K2Cl symport system in the TAL reabsorbs the solutes

87
Q
  1. Regarding the Aldoterone Paradox, which of the following statements best explains the concept? Aldosterone increases when:

A. Serum potassium increases
B. Serum sodium increases
C. Plasma osmolarity increases
D. RAAS is activated

A

Serum potassium increases

88
Q
  1. Increasing the blood flow in the vasa recta will lead to which of the following consequences?

A. Osmolarity of the medullary interstitium increases

B. The hyperosmolar medullary interstitium will be prolonged

C. Plasma osmolarity along the ascending limb of the vasa recta increases

D. The countercurrent multiplier activity increases

A

Plasma osmolarity along the ascending limb of the vasa recta increases

89
Q
  1. Vasoconstriction of the efferent arteriole will cause the countercurrent exchanger to

A. Prolonged hyperosmolarity of the medullary interstitium

B. Increase the reabsorption of urea more than Na+

C. Increase the hydrostatic pressure in its ascending limb

D. Increase the reabsorption of Na+ more than urea

A

Increase the reabsorption of Na+ more than urea

90
Q
  1. Which of the following is NOT an effect of the 1Na+-1K+-2Cl transporter in the apical membrane of the TAL?

A. It moves K+ against its gradient into the cell

B. It decreases the osmolality of the tubular fluid of the TAL

C. It maintains electroneutrality by moving 2 positively charged solutes with 2 molecules of a negatively charged solute

D. It enhances urea absorption in the tubular fluid

A

It maintains electroneutrality by moving 2 positively charged solutes with 2 molecules of a negatively charged solute

91
Q
  1. A patient reports he is always thirsty, and his breath smells like acetone. He is suspected to have diabetes mellitus, and this is confirmed by a urine sample that tests positive for glucose and a blood sample that shows an FBS of 400mg/dL. Compared with normal, one would expect these changes in the urine.

A. Decreased urine pH, increased NH4+ excretion, decreased 24hr urine volume, decreased renal HCO3 production

B. Increased urine pH, decreased NH4+ excretion, decreased 24hr urine volume, decreased renal HCO3 production

C. Decreased urine pH, increased NH4+ excretion, increased 24hr urine volume, increased renal HCO3 production

D. Decreased urine pH, decreased NH4+ excretion, decreased 24hr urine volume, decreased renal HCO3 production

A

Decreased urine pH, decreased NH4+ excretion, decreased 24hr urine volume, decreased renal HCO3 production

92
Q
  1. A concentrated urine can be produced when the vasa recta causes which of the choices below?

A. A decrease in water reabsorption in the ascending limb of the vasa recta

B. An increase in NaCl reabsorption in ascending limb of the vasa recta

C. A decrease in the resistance in the descending and ascending limbs

D. A decrease in the plasma osmolarity of ascending limb of the vasa recta

A

An increase in NaCl reabsorption in ascending limb of the vasa recta

93
Q
  1. Acute metabolic acidosis tends to _____

intracellular K+ concentration and _____ K+ secretion by the cortical collecting tubules
A. Decrease; increase
B. Decrease; decrease
C. Increase; decrease
D. Increase; increase

A

Decrease; decrease

94
Q
  1. SGK1 or serum glucocorticoid-stimulated kinase enhances sodium reabsorption because it increases the activity of which of the choices below?

A. Potassium secretion across the apical membrane
B. Na/H exchanger activity
C. Epithelial sodium channels
D. Na/K ATPase activity

A

Na/K ATPase activity

95
Q
  1. This will occur in LETTER D when the oncotic pressure in LETTER C (Picture)
    A. Increased tubular fluid volume
    B. Decreased tubular fluid osmolarity

C. Increased tubular fluid oncotic pressure
D. Increased tubular fluid reabsorption

A

Increased tubular fluid oncotic pressure

96
Q
  1. Referring to the drawing, an increase in perfusion will lead to an increase in filtration fraction where?

A. B
B. C
C. A
D. D

A

A

97
Q
  1. Referring to the drawing, the type of capillary network in LETTER B
    A. Discontinuous
    B. Fenestrated
    C. Sinusoidal
    D. Continuous
A

Fenestrated

98
Q
  1. Referring to the drawing, when the macula densa reabsorbs a high amount of sodium chloride, tubuloglomerular feedback will cause the vessel diameter of LETTER ___ to change

A. B
B. C

C. D
D. A

A

A

99
Q
  1. Referring to the drawing, during euvolemia, when letter D reabsorbs 67% of solutes and water. This will make the osmolarity in LETTER ___
    A. B
    B. D
    C. A
    D. C
A

B

100
Q
  1. Glomerular filtration produces an infiltrate of plasma

A. In which the concentration of protein is equal to plasma

B. Containing only those substances which must be eliminated in the urine

C. In which the concentration of electrolytes is equal to plasma
D. That has a higher concentration of electrolytes

A

In which the concentration of electrolytes is equal to plasma

101
Q

The plasma concentration at which a particular substance begins to appear in the urine
A. Transport maximum

B. Fractional excretion

C. Renal Threshold

D. Filtered load

A

Renal Threshold

102
Q

Which of the following might you expect to find associated with chronic renal failure?
A. A low plasma creatinine concentration

B. Low PTH

C. A high hematocrit

D. Decrease bone density

A

Decrease bone density

103
Q

2 Which of the following best describes the vasa recta? The vasa recta:

A. Extend from the efferent arterioles of the superficial cortical nephrons
B. Are straight portions of the proximal tubules
C. Comprise the capillary network in the medullary region of the kidney
D. Are the straight segments of the arterioles found near the macula densa

A

Comprise the capillary network in the medullary region of the kidney

104
Q
  1. The renal countercurrent mechanism is dependent upon the relationship between the:
    A. Loop of Henle and the macula densa
    B. Distal tubule and the macula densa
    C. Loop of Henle and the vasa recta
    D. Glomerulus and the afferent and efferent arterioles
A

Loop of Henle and the vasa recta

105
Q
  1. An increase in the osmolality of the ECF will:
    A. Stimulate the volume and osmoreceptors and stimulate
    ADH

C. Inhibit the volume and osmoreceptors and inhibit
ADH

D. Stimulate the volume and osmoreceptors and inhibit
ADH

E. Inhibit the volume and osmoreceptors and stimulate
ADH

A

Stimulate the volume and osmoreceptors and inhibit
ADH

106
Q

Which of the following tends to increase potassium secretion by the cortical collecting tubule?
A. A diuretic that decreases loop of Henle sodium reabsorption (e.g., furosemide)
B. Decreased plasma potassium concentration
C. Low sodium intake

D. Acute metabolic acidosis

A

A diuretic that decreases loop of Henle sodium reabsorption (e.g., furosemide)

107
Q

Which of the following tends to increase GFR?
A. Increased Bowman’s capsule hydraulic pressure
B. Decreased efferent arteriolar resistance
C. Increased afferent arteriolar resistance
D. Increased glomerular capillary filtration coefficient

A

Increased glomerular capillary filtration coefficient

108
Q

1 The maximum clearance rate possible for a substance that is completely cleared from the plasma is equal to which of the following?
A. GFR

B. Renal plasma flow

C. Filtration fraction

D. Filtered load of that substance

A

Renal plasma flow

109
Q
  1. A patient was admitted to the hospital because of glomerulonephritis. His GFR was assessed to be at 50%. You expect to see a high plasma concentration of which of the following?

A. Glucose

B. Creatinine

C. Sodium

D. Potassium

A

Creatinine

110
Q

Under conditions of normal renal function, the concentration of urea in the tubular fluid at the tip of the 2nd half of the proximal tubule is expected to be:
A. Higher in the urine

B. Higher than plasma urea

C. Higher than the tubular fluid concentration of urea at the tip of the loop of Henle
D. Lower than plasma urea because urea is absorbed in the CCD

A

Higher than plasma urea

111
Q
  1. In normal kidneys, which if the following is true of the renal tubular fluid that flows through the early DCT near the macula densa?
    A. Hypertonic compared with plasma
    B. Isotonic compared with plasma
    C. Hypotonic compared with plasma
    D. Hypertonic compared with plasma during dehydration
A

Hypotonic compared with plasma

112
Q

A kidney donor had a renal arteriogram which revealed renal artery stenosis with a GFR of only 25% of his remaining kidney. Which of the following changes do you expect to occur?
A. Decreased urinary creatinine excretion at 25% of normal
B. Decreased urinary sodium excretion at 25% of normal
C. Increased serum creatinine to about 4x of normal
D. Increased serum sodium concentration

A

Increased serum creatinine to about 4x of normal

113
Q

When a vasodilator is administered to a normal individual that leads to a 50% decrease in arteriolar resistance, but no effect on blood pressure, you expect that there will be a:
A. Increase in RBF, GFR and a decrease in peritubular capillary hydrostatic pressure
B. Increase in RBF, GFR and peritubular capillary hydrostatic pressure
C. Decrease in RBF, GFR and an increase in peritubular capillary hydrostatic pressure
D. Decrease in RBF, GFR and peritubular capillary hydrostatic pressure

A

Increase in RBF, GFR and peritubular capillary hydrostatic pressure

114
Q

Which of the following changes tends to increase peritubular capillary fluid reabsorption?
A. A low filtration fraction
B. A high blood pressure

C. A high efferent arteriolar resistance
D. A high renal blood flow

A

A high efferent arteriolar resistance

115
Q
  1. Intravenous infusion of 1 liter 0.45% NaCl will lead to a decrease of which of the following after osmotic equilibrium?
    a. There will be no change

b. Intracellular fluid volume

c. Intracellular fluid osmolarity

d. Extracellular fluid volume

A

Intracellular fluid osmolarity

116
Q
  1. When the dietary intake of potassium increases, K+ balance is maintained in the kidneys via which logical mechanism?
    a. Decreased K+ reabsorption by the TAL
    b. Decreased K+ reabsorption in the PCT
    c. Decreased K+ reabsorption by the late DCT and CCD
    d. Decreased glomerular filtration of K+
A

Decreased K+ reabsorption by the TAL

117
Q

As a primary care physician, you prescribed a loop diuretic to your patient for 1 month to manage his hypertension. A diuretic promotes urination. Which of the following will you expect to find once the patient comes back to you after a month?
A. A low ECF volume

B. A low plasma potassium

C. A normal urine solutions

D. A low blood pressure

A

A low plasma potassium

118
Q
  1. As hydrostatic pressure in the paracellular spaces of the PCT increases, which of the following choices will occur?
    a. Osmotic pressure in the paracellular spaces increases
    b. Water moves towards the tubular fluid
    c. Hydrostatic pressure in the tubular fluid decreases
    d. Moves towards the capillaries
A

Hydrostatic pressure in the tubular fluid decreases

119
Q

Which of the following concerning the glomerular filtration rate (GFR), is correct? It:
A. Is independent of the size of the renal capillary bed
B. Is approximately 250ml/min

C. Is maintained despite a fall in systemic pressure below 90mHg
D. Exceeds the clearance of a substance if there is net tubular reabsorption

A

Exceeds the clearance of a substance if there is net tubular reabsorption

120
Q
  1. A distraught 26 year old male, came to the ER, and his ABG revealed pH
    7.46, CO2 32mmHg, HCO3-23mEq/L. Which of the following would be the acid base condition?
    a. Combined, Uncompensated

b. Respiratory alkalosis, Uncompensated
c. Metabolic alkalosis, Uncompensated
d. Respiratory acidosis, Partially compensated

A

Respiratory alkalosis, Uncompensated

121
Q

A 54 yr old female, diabetic, hypertensive was sent to the hospital, and her ABG showed the following - ph 7.31, CO2 44mmHg, HCO3- 20 mEq/L/ Which acid base disorder would be likely?

A. Metabolic acidosis, Uncompensated
B. Respiratory alkalosis, partially compensated
C. Respiratory acidosis, Uncompensated
D. Metabolic acidosis, partially compensated

A

Metabolic acidosis, Uncompensated

122
Q
  1. A respiratory alkalosis is a primary acid-base disorder in which arterial?
    a. pCO2 falls to a level lower than expected

b. HCO3 is normal

c. HCO3 is low
d. pCO2 falls to a level higher than expected

A

pCO2 falls to a level lower than expected

123
Q
  1. A decrease in plasma bicarbonate can be caused by two mechanisms:
    a. A gain of strong acid and loss of base
    b. A gain of both base and acid

c. A gain of base and a loss of acid
d. A gain of base and again of ammonia

A

A gain of strong acid and loss of base

124
Q
  1. In hyperchloremic metabolic acidosis, you expect the anion gap to be:
    a. Normal

b. Low

c. High

d. Initially high

A

Normal